Những câu hỏi liên quan
Pham Trong Bach
Xem chi tiết
Cao Minh Tâm
13 tháng 7 2017 lúc 3:55

Giải bài tập Toán 12 | Để học tốt Toán 12

Xét tứ diện đều A.BCD cạnh bằng a. Gọi G 1 , G 2 , G 3   v à   G 4  lần lượt là tâm của các tam giác BCD, ACD, ABD và ABC.

Gọi M là trung điểm của BC.

Xét tam giác AMD có:

Giải bài tập Toán 12 | Để học tốt Toán 12

 

 

Bình luận (0)
chảnh chó gì cái dkm nhà...
Xem chi tiết
Nguyễn Thắng Tùng
7 tháng 2 2016 lúc 14:22

1)

Chia lăng trụ ABD.A'B'D' thành ba tứ diện DABD', A'ABD', A'B'BD'. Phép đối xứng qua (ABD') biến DABD' thành A'ABD', Phép đối xứng qua (BA'D') biến A'ABD' thành A'B'BD' nên ba tứ diện DABA', A'ABD', A'B'BD' bằng nhau

Làm tương tự đối với lăng trụ BCD.B'C'D' ta sẽ chia được hình lập phương thành sáu tứ diện bằng nhau.


 

Bình luận (0)
chảnh chó gì cái dkm nhà...
Xem chi tiết
chảnh chó gì cái dkm nhà...
7 tháng 2 2016 lúc 14:15

ai giải được thì tik 6 nghĩa là 2 ngày nha

Bình luận (0)
Nguyễn Xuân Sáng
7 tháng 2 2016 lúc 14:42

1)

Chia lăng trụ ABD.A’B’D’ thành ba tứ diện DABD’, A’ABD’, A’B’BD’. Phép đối xứng qua (ABD’) biến DABD’ thành A’ABD’, Phép đối xứng qua (BA’D’) biến A’ABD’ thành A’B’BD’ nên ba tứ diện DABA’, A’ABD’, A’B’BD’ bằng nhau.

Làm tương tự đối với lăng trụ BCD.B’C’D’ ta sẽ chia được hình lập phương thành sáu tứ diện bằng nhau.

Bình luận (0)
Pham Trong Bach
Xem chi tiết
Cao Minh Tâm
29 tháng 1 2019 lúc 15:22

Đáp án C.

Đặt (H) là hình tứ diện đều ABCD, cạnh bằng A. Gọi E ; F ; I ; J  lần lượt là tâm của các mặt  A B C ; A B D ; A C D ; B C D   .

Kí hiệu như hình vẽ.

Ta có M E M C = M F M D = 1 3 ⇒ E F C D = 1 3 ⇒ E F = C D 3 = a 3 .

Vậy tứ diện  là tứ diện đều có cạnh bằng a 3 .

Tỉ số thể tích của diện tích toàn phần tứ diện đều  và tứ diện đều ABCD là  a 3 a 2 = 1 9

Bình luận (0)
Pham Trong Bach
Xem chi tiết
Cao Minh Tâm
25 tháng 1 2019 lúc 17:56

Đáp án A

Bình luận (0)
Nguyễn Khánh Lâm
1 tháng 12 2023 lúc 19:07

[Chính Hãng] Gói thẻ bài One Piece 05 Awakening Of The New Era Booster Pack Bandai Card Game | Shopee Việt Nam

Bình luận (0)
Pham Trong Bach
Xem chi tiết
Cao Minh Tâm
23 tháng 1 2017 lúc 6:22

Đáp án A

Bình luận (0)
Pham Trong Bach
Xem chi tiết
Cao Minh Tâm
7 tháng 7 2017 lúc 13:04

Chọn B.

Nếu có một mặt cầu ngoại tiếp lăng trụ thì đáy của lăng trụ phải nội tiếp trong một đường tròn, điều này không đúng cho tứ giác lồi bất kì.

Bình luận (0)
Pham Trong Bach
Xem chi tiết
Cao Minh Tâm
22 tháng 3 2019 lúc 7:12

Đáp án A.

Có tất cả 15 điểm được tô màu gồm 4 đỉnh của tứ diện, 6 trung điểm của 6 cạnh, 4 trọng tâm của 4 mặt bên và 1 trọng tâm của tứ diện.

Không gian mẫu là “Chọn ngẫu nhiên 4 trong số 15 điểm đã tô màu”. Số phần tử của không gian mẫu là  n Ω = C 15 4   .

Gọi A là biến cố “4 điểm được chọn đồng phẳng”. Suy ra  là biến cố “4 điểm được chọn là 4 đỉnh của một hình tứ diện”. Để xác định số kết quả thuận lợi cho biến cố A ta xét các trường hợp sau:

a. 4 điểm cùng thuộc “một mặt bên của tứ diện”

Một mặt bên có 7 điểm được tô màu nên số cách chọn 4 điểm (đồng phẳng) trên một mặt bên là C 7 4  (cách).

Có tất cả 4 mặt bên nên số cách chọn thỏa mãn trường hợp a. là 4. C 7 4  (cách).

b. 4 điểm cùng thuộc mặt phẳng “chứa 1 cạnh của tứ diện và trung điểm của cạnh đối diện:.

Mặt phẳng đó có 7 điểm được tô màu nên số cách chọn 4 điểm (đồng phẳng) trên mỗi mặt là C 7 4  (cách).

Hình tứ diện có 6 cạnh nên có tất cả 6 mặt như thế. Số cách chọn 4 điểm thỏa mãn trường hợp b. là 6 C 7 4  (cách).

c. 4 điểm cùng thuộc mặt phẳng “chứa 1 đỉnh và đường trung bình của tam giác đối diện đỉnh đó”.

Mặt phẳng đó có 5 điểm được tô màu nên số cách chọn 4 điểm (đồng phẳng) trên mỗi mặt là C 5 4  (cách).

Do mỗi mặt bên là một tam giác có 3 đường trung bình, nên mỗi đỉnh có tương ứng 3 mặt phẳng như thế (chứa đỉnh và đường trung bình). Mà tứ diện có 4 đỉnh nên có tất cả   3.4 = 12 mặt phẳng ở trường hợp c.

Vậy số cách chọn thỏa mãn trường hợp c. là   12 C 5 4 (cách).

d. 4 điểm cùng thuộc mặt phẳng “chứa 2 đường nối 2 trung điểm của các cạnh đối diện”.

Có 3 đường nối 2 trung điểm của các cạnh đối diện. Số mặt phẳng được tạo thành từ 2 trong 3 đường đó là   C 3 2 (mặt phẳng).

Mỗi mặt phẳng như thế có 5 điểm được tô màu nên số cách chọn 4 điểm (đồng phẳng) là   C 5 4 (cách).

Vậy số cách chọn thỏa mãn trường hợp d. là C 3 2 . C 5 4   (cách).

Số kết quả thuận lợi cho biến cố A là  n A = 4 C 7 4 + 6 C 7 4 + 12 C 5 4 + C 3 2 . C 5 4 = 425   .

Vậy xác suất cần tính là

P A ¯ = 1 − P A = 1 − n A n Ω = 1 − 425 C 15 4 = 188 173

Bình luận (0)
Nguyễn Nhật
Xem chi tiết